LSAT and Law School Admissions Forum

Get expert LSAT preparation and law school admissions advice from PowerScore Test Preparation.

 Administrator
PowerScore Staff
  • PowerScore Staff
  • Posts: 8917
  • Joined: Feb 02, 2011
|
#81132
Complete Question Explanation

Resolve the Paradox, Except. The correct answer choice is (B).

Answer choice (A):

Answer choice (B): This is the correct answer choice.

Answer choice (C):

Answer choice (D):

Answer choice (E):

This explanation is still in progress. Please post any questions below!
 Pragmatism
  • Posts: 68
  • Joined: Jan 11, 2018
|
#45849
I narrowed it down to B and E,but I ended up going with E. I want gauge your assessment on my thinking to choosing E over B.
Answer choice B common drugs are priced less than not so common, which would make sense because the turn over rate for common drugs is high thus leaving the price point high in bulk orders would be counterintuitive, whereas the not so common are made for a niche market, which even at bulk would pay that because they have a specific market for it.

Answer choice E says that they are tailored to individuals doctors who want “relativity small quantity.” Well, a discount is warranted for any bulk order if the supplier is a wholesaler, but what lead me to pick this over B, the right answer, was the fact that B gave my notion of supply and demand greater credence for justification than answer choice E.

Please give me your input.

Thanks
 Alex Bodaken
PowerScore Staff
  • PowerScore Staff
  • Posts: 136
  • Joined: Feb 21, 2018
|
#45865
Pragmatism,

Thanks for the question. Let me see if I can help.

This is a "resolve the paradox" EXCEPT question, meaning that the 4 incorrect answer choices will help to resolve the paradox, while the 1 correct answer choice will not.

Looking at answer choice (E) first...it reads "The prices suggested by the independently published pharmaceutical price guidebook are for sales of relatively small quantities of pharmaceutical drugs to individual doctors." If this were true, would it help to resolve the paradox? It would, because if the guidebook is publishing prices for smaller quantities of sales than the leading wholesale sellers are selling, then the prices in the book and from the leading sellers are no longer directly comparable, thereby helping to resolve the paradox.

Answer choice (B) reads: "Suggested wholesale prices for the most common pharmaceutical drugs tend to be less than those for less common pharmaceutical drugs." This doesn't help us to resolve the paradox because the stimulus tells us that we are comparing the same drugs from the leading sellers and in the guidebook; therefore, the differences between the prices of the most common and less common drugs is irrelevant. Because it doesn't help to resolve the paradox, answer choice (B) is credited here.

I want to add one other note of caution here: in explaining your reasoning, you used a lot of what I think could be considered outside knowledge (supply and demand, etc.) to determine the correct answer. Be very careful doing that: generally, the LSAT expects you to have no outside knowledge, and you should be able to deduce the answer from the information provided (as we can here). Often, bringing in outside knowledge can actually be confusing, and can lead us away from the answer choice that can be gotten through analysis of the stimulus and question stem.

Hope that helps!
Alex
 Pragmatism
  • Posts: 68
  • Joined: Jan 11, 2018
|
#45947
Thank you so much for that.

Get the most out of your LSAT Prep Plus subscription.

Analyze and track your performance with our Testing and Analytics Package.